LSAT and Law School Admissions Forum

Get expert LSAT preparation and law school admissions advice from PowerScore Test Preparation.

User avatar
 Dave Killoran
PowerScore Staff
  • PowerScore Staff
  • Posts: 5850
  • Joined: Mar 25, 2011
|
#43446
Complete Question Explanation
(The complete setup for this game can be found here: lsat/viewtopic.php?t=16406)

The correct answer choice is (D)

Answer choice (A) is incorrect because H is in group 1 but L is not, a violation of the third rule.

Answer choice (B) is incorrect because F and J are not in the same group.

Answer choice (C) is incorrect because G and M are in group 2 together, a violation of the second rule.

Answer choice (D) is the correct answer choice.

Answer choice (E) is incorrect because G and M are in group 1 together, a violation of the second rule.
 bwise2928
  • Posts: 3
  • Joined: Oct 19, 2021
|
#93222
Why can't A be correct? Couldn't it play out GFJL in Group 1 and KNMH in Group 2? Why does H have to be in group 1?
 bwise2928
  • Posts: 3
  • Joined: Oct 19, 2021
|
#93223
bwise2928 wrote: Mon Jan 10, 2022 6:57 pm Why can't A be correct? Couldn't it play out GFJL in Group 1 and KNMH in Group 2? Why does H have to be in group 1?
Never mind, just realized it was an MBT

Get the most out of your LSAT Prep Plus subscription.

Analyze and track your performance with our Testing and Analytics Package.